I don't understand how to laplace transform heaviside functions

Click For Summary
The discussion revolves around understanding how to apply the Laplace transform to the function (6-t)heaviside(t-2). The user expresses confusion about the application of the Laplace transform rules for Heaviside functions, particularly regarding the transformation of shifted functions. A key point made is that to correctly transform f(t)heaviside(t-c), one should express f(t) in terms of (t-c) and then apply the transformation rules. The user realizes that they were incorrectly transforming the shifted function and receives advice on the correct approach, which involves substituting and transforming accordingly. The conversation highlights the importance of properly handling shifted functions in Laplace transforms.
1MileCrash
Messages
1,338
Reaction score
41

Homework Statement



(6-t)heaviside(t-2)

This is just one term of the real problem I'm working, but it will serve to help me figure this out.

Homework Equations





The Attempt at a Solution



http://www.wolframalpha.com/input/?i=laplace+transform+{(6-t)heaviside(t-2)}

I really, REALLY don't understand how to do this, at all.

My textbook makes the following claims:

heaviside(t-c) laplace transform: e^(-c)/s
heaviside(t-c)f(t-c) laplace transform: e^(-cs)F(s) where F denotes the transform of f

Ok, fine. Apparently I am not very math literate because that makes me do the following:

(6-t)heaviside(t-2) = (4-(t-2))heaviside(t-2) = 4heaviside(t-2) - (t-2)heaviside(t-2)

By the rules above, the laplace transform I want to say is:

4e^(-2s)/s - e^(-2s)(1/s^2 - 2/s)


Which is wrong. I have NO idea what to do with this, I don't understand what these rules are telling me to do.


Thanks for any advice.
 
Last edited:
Physics news on Phys.org
*sigh* nevermind, I see that I am transforming the shifted function and I shouldn't be.

This is annoying.
 
1MileCrash said:

Homework Statement



(6-t)heaviside(t-2)

This is just one term of the real problem I'm working, but it will serve to help me figure this out.

Homework Equations


The Attempt at a Solution



http://www.wolframalpha.com/input/?i=laplace+transform+{(6-t)heaviside(t-2)}

I really, REALLY don't understand how to do this, at all.

My textbook makes the following claims:

heaviside(t-c) laplace transform: e^(-c)/s
heaviside(t-c)f(t-c) laplace transform: e^(-cs)F(s) where F denotes the transform of f

Ok, fine. Apparently I am not very math literate because that makes me do the following:

(6-t)heaviside(t-2) = (4-(t-2))heaviside(t-2) = 4heaviside(t-2) - (t-2)heaviside(t-2)

By the rules above, the laplace transform I want to say is:

4e^(-2s)/s - e^(-2s)(1/s^2 - 2/s)Which is wrong. I have NO idea what to do with this, I don't understand what these rules are telling me to do.

Thanks for any advice.

I will use ##u(t)## for heaviside(t). You have the formula$$
\mathcal L(f(t-c)u(t-c) = e^{-sc}\mathcal L(f(t))=e^{-sc}F(s)$$The problem with using this for taking transforms is that usually you are asked to take the transform of ##f(t)u(t-c)##. In your example where ##f(t) = 6 -t## that is why they jump through hoops to write ##6-t =4-(t-2)##, so it is written as a function of ##(t-2)## to match the ##u(t-2)##.

Let's look at what you would get taking the transform of ##f(t)u(t-c)##:$$
\mathcal L(f(t)u(t-c) = \int_0^\infty e^{-st}f(t)u(t-c)\, dt =\int_c^\infty e^{-st}f(t)\, dt$$Now let ##v = t-c## so we get$$
\int_0^\infty e^{-s(v+c)}f(v+c)\, dv =e^{-sc}\int_0^\infty e^{-sv}f(v+c)\, dv =
e^{-sc}\int_0^\infty e^{-st}f(t+c)\, dt=e^{-sc}\mathcal L(f(t+c))$$This says that if you want to transform ##f(t)u(t-c)## you can do the following steps:
1. Replace ##t## by ##t+c## in the formula for ##f(t)##.
2. Transform that.
3. Multiply the result by ##e^{-sc}##.
Try it and see if you like it.

[Edit] I see you solved it while I was typing. Try my suggestion anyway, you might like it better.
 
Question: A clock's minute hand has length 4 and its hour hand has length 3. What is the distance between the tips at the moment when it is increasing most rapidly?(Putnam Exam Question) Answer: Making assumption that both the hands moves at constant angular velocities, the answer is ## \sqrt{7} .## But don't you think this assumption is somewhat doubtful and wrong?

Similar threads

  • · Replies 1 ·
Replies
1
Views
1K
  • · Replies 10 ·
Replies
10
Views
2K
Replies
3
Views
2K
  • · Replies 4 ·
Replies
4
Views
3K
  • · Replies 3 ·
Replies
3
Views
3K
  • · Replies 7 ·
Replies
7
Views
2K
  • · Replies 3 ·
Replies
3
Views
3K
  • · Replies 3 ·
Replies
3
Views
5K
  • · Replies 13 ·
Replies
13
Views
4K
  • · Replies 1 ·
Replies
1
Views
943